Why is that bb so non-trivial?

I found this interesting (to me, at least) when I was solving this problem:

"Let a1,a2,...,an,b,ka_1,a_2,...,a_n,b,k be positive reals so that a1+a2+...+annka_1+a_2+...+a_n \le nk. Prove that: (a1+b)(a2+b)...(an+b)(k+b)n(a_1+b)(a_2+b)...(a_n+b) \le (k+b)^n ."

At first, my thought was: "Oh, so it's obvious that kna1a2...ank^n \ge a_1*a_2* ... *a_n. But what about the bb? There's no way to throw it away." And right now, I'm stuck at that exact point. Reverse Rearrangement doesn't seem to work, and there is a "seemingly" trivial solution using Buffalo Way, but I want to find a "better" way than just plug-and-bash.

So, my question is: Is there a "good-looking" way to solve the problem? If there is, what is your inspiration?

Any help would be greatly appreciated. Thanks for reading!

P/S: If you can find any counterexamples, please tell me immediately and I will delete/edit this note.

#Algebra

Note by Steven Jim
3 years, 10 months ago

No vote yet
1 vote

  Easy Math Editor

This discussion board is a place to discuss our Daily Challenges and the math and science related to those challenges. Explanations are more than just a solution — they should explain the steps and thinking strategies that you used to obtain the solution. Comments should further the discussion of math and science.

When posting on Brilliant:

  • Use the emojis to react to an explanation, whether you're congratulating a job well done , or just really confused .
  • Ask specific questions about the challenge or the steps in somebody's explanation. Well-posed questions can add a lot to the discussion, but posting "I don't understand!" doesn't help anyone.
  • Try to contribute something new to the discussion, whether it is an extension, generalization or other idea related to the challenge.
  • Stay on topic — we're all here to learn more about math and science, not to hear about your favorite get-rich-quick scheme or current world events.

MarkdownAppears as
*italics* or _italics_ italics
**bold** or __bold__ bold

- bulleted
- list

  • bulleted
  • list

1. numbered
2. list

  1. numbered
  2. list
Note: you must add a full line of space before and after lists for them to show up correctly
paragraph 1

paragraph 2

paragraph 1

paragraph 2

[example link](https://brilliant.org)example link
> This is a quote
This is a quote
    # I indented these lines
    # 4 spaces, and now they show
    # up as a code block.

    print "hello world"
# I indented these lines
# 4 spaces, and now they show
# up as a code block.

print "hello world"
MathAppears as
Remember to wrap math in \( ... \) or \[ ... \] to ensure proper formatting.
2 \times 3 2×3 2 \times 3
2^{34} 234 2^{34}
a_{i-1} ai1 a_{i-1}
\frac{2}{3} 23 \frac{2}{3}
\sqrt{2} 2 \sqrt{2}
\sum_{i=1}^3 i=13 \sum_{i=1}^3
\sin \theta sinθ \sin \theta
\boxed{123} 123 \boxed{123}

Comments

If you've proven kna1a2ank^n \ge a_1 a_2 \ldots a_n for any a1,a2,,an,ka_1, a_2, \ldots, a_n, k, substitute kk+bk \gets k+b and aiai+ba_i \gets a_i+b and you're done.

Another method: aim to maximize (a1+b)(a2+b)(an+b)(a_1+b) (a_2+b) \ldots (a_n+b). Prove that if ai<nk\sum a_i < nk then this product is not maximized. Prove that if there is any aiaja_i \neq a_j then this product is also not maximized. So the maximum of the product is achieved when ai=ka_i = k for all ii, done.

Ivan Koswara - 3 years, 10 months ago

Log in to reply

That is actually the one I've been thinking about: I can't guarantee that the substitution will fit. If I could then I think this problem would've been too easy as it's so trivial.

Steven Jim - 3 years, 10 months ago

Log in to reply

It works. Prove the special case kna1a2ank'^n \ge a'_1 a'_2 \ldots a'_n, and use it as a lemma.

Ivan Koswara - 3 years, 10 months ago

By AM-GM, (a1+b)(a2+b)(an+b)n(a1+b)+(a2+b)++(an+b)n=a1+a2++ann+bk+b. \begin{aligned} \sqrt[n]{(a_1 + b)(a_2 + b) \dotsm (a_n + b)} &\le \frac{(a_1 + b) + (a_2 + b) + \dots + (a_n + b)}{n} \\ &= \frac{a_1 + a_2 + \dots + a_n}{n} + b \\ &\le k + b. \end{aligned} Therefore, (a1+b)(a2+b)(an+b)(k+b)n.(a_1 + b)(a_2 + b) \dotsm (a_n + b) \le (k + b)^n.

Jon Haussmann - 3 years, 10 months ago

In other words, direct AM-GM works.

You might be thinking of another problem, namely (ai+b)(ain+b)n \prod ( a_i + b ) \leq ( \sqrt[n]{\prod a_i} + b ) ^n .

Calvin Lin Staff - 3 years, 10 months ago

Log in to reply

Thanks for mentioning.

Steven Jim - 3 years, 10 months ago

What is "Buffalo way"? Is this some new inequality trick?

Pi Han Goh - 3 years, 10 months ago

Log in to reply

Here's the link.

Steven Jim - 3 years, 10 months ago
×

Problem Loading...

Note Loading...

Set Loading...